0% found this document useful (0 votes)
76 views7 pages

UPSC Civil Services Main 1986 - Mathematics Calculus: Sunder Lal

This document contains the solutions to questions from the UPSC Civil Services Main 1986 exam on the topic of mathematics. The first question examines the differentiability of a piecewise defined function f(x) at x=0. It is shown that f(x) is differentiable at x=0 with f'(0)=0. The second part of the first question evaluates the limit of an integral expression involving f(x) and obtains the value -1/2. The second question uses Rolle's theorem to establish an inequality relationship between an-bn and nbn-1(a-b) where a>b and n>1. The third question evaluates the expression x2

Uploaded by

sayhigaurav07
Copyright
© © All Rights Reserved
We take content rights seriously. If you suspect this is your content, claim it here.
Available Formats
Download as PDF, TXT or read online on Scribd
0% found this document useful (0 votes)
76 views7 pages

UPSC Civil Services Main 1986 - Mathematics Calculus: Sunder Lal

This document contains the solutions to questions from the UPSC Civil Services Main 1986 exam on the topic of mathematics. The first question examines the differentiability of a piecewise defined function f(x) at x=0. It is shown that f(x) is differentiable at x=0 with f'(0)=0. The second part of the first question evaluates the limit of an integral expression involving f(x) and obtains the value -1/2. The second question uses Rolle's theorem to establish an inequality relationship between an-bn and nbn-1(a-b) where a>b and n>1. The third question evaluates the expression x2

Uploaded by

sayhigaurav07
Copyright
© © All Rights Reserved
We take content rights seriously. If you suspect this is your content, claim it here.
Available Formats
Download as PDF, TXT or read online on Scribd
You are on page 1/ 7

UPSC Civil Services Main 1986 - Mathematics

Calculus
Sunder Lal
Retired Professor of Mathematics
Panjab University
Chandigarh

January 16, 2010

Question 1(a) 1. A function f (x) is defined as follows:


( 1
e1 x2 sin x1 , x 6= 0
f (x) =
0, x=0

Examine whether or not f (x) is differentiable at x = 0.

2. If f 0 (x) exists and is continuous, find the value of

1 x
Z
lim (x 3y)f (y) dy
x0 x2 0

Solution.
f (x) f (0)
1. By definition, f 0 (0) = lim , if this limit exists.
x0 x
f (x) f (0) 1 1 1
lim = lim e1 x2 sin
x0 x x0 x x
1 12 1
= e lim 2 e x lim x sin
x0 x x0 x
1 1
= e lim tet 0 Letting t = 2
, also lim x sin = 0
t x x0 x
= 0 lim tet = 0
t

Thus f (x) is differentiable at x = 0, and f 0 (0) = 0.

1
2. Define
Z x
F (x) = (x 3y)f (y) dy
0
Z x Z x
= x f (y) dy 3 yf (y) dy
0 0
Z x Z x
0
F (x) = f (y) dy + xf (x) 3xf (x) = f (y) dy 2xf (x)
0 0
F 00 (x) = f (x) 2f (x) 2xf 0 (x) = f (x) 2xf 0 (x)
Note that F 00 (x) exists because f 0 (x) exists.
F (x)
The required limit is lim 2 . Since lim F (x) = 0, lim x2 = 0 and both are differen-
x0 x x0 x0
tiable, we can apply LHospitals rule to get
F (x) F 0 (x)
lim = lim
x0 x2 x0 2x

Again LHospitals rule applies, so


F (x) F 00 (x) f (x) 2xf 0 (x) f (0)
lim 2
= lim = lim =
x0 x x0 2 x0 2 2
since f 0 (x) exists and is continuous, so limx0 xf 0 (x) = 0.
Thus Z x
1 f (0)
lim 2 (x 3y)f (y) dy =
x0 x 0 2

Question 1(b) Use Rolles theorem to establish that under suitable conditions (to be stated)

f (a) f (b) f (a) f 0 ()
g(a) g(b) = (b a) g(a) g 0 () , a < < b

Hence or otherwise deduce the inequality


nbn1 (a b) < an bn < nan1 (a b)
where a > b and n > 1.
Solution. Let f (x), g(x) be continuous in the closed interval [a, b] and differentiable in the
open interval (a, b). Let

f (a) f (x) x a f (a) f (b)
F (x) =
g(a) g(x) b a g(a) g(b)
xa
= [f (a)g(x) g(a)f (x)] [f (a)g(b) g(a)f (b)]
ba
Thus F (x) is

2
1. continuous in the closed interval [a, b].
2. differentiable in the open interval (a, b)
3. F (a) = F (b) = 0.
Thus F (x) satisfies the requirements of Rolles theorem, consequently, there exists , a <
< b such that F 0 () = 0. But
1
F 0 () = [f (a)g 0 () g(a)f 0 ()] [f (a)g(b) g(a)f (b)]
ba
f (a) f 0 ()
1 f (a) f (b) = 0

= 0
g(a) g () b a g(a) g(b)

therefore
f (a) f (b) 0
= (b a) f (a) f 0 () , a < < b


g(a) g(b) g(a) g ()
Let f (x) = xn , g(x) = 1, then the above result implies
n n n
b a b n n1
1 1 = (a b) 1
, b < < a
0
or an bn = (a b)n n1 . Now since bn1 < n1 < an1 , we get
nbn1 (a b) < an bn < nan1 (a b), as required.
(ax3 + by 3 )n 2u 2u 2u
Question 1(c) If u = + xf ( xy ), find the value of x2 2 + 2xy + y2 2 .
3n(3n 1) x x y y
Solution. We use the result proved in 2006, question 2(b): If f (x, y) is a homogeneous
function of degree n possessing continuous partial derivatives of degree 2,
2f 2f 2
2 f
x2 + 2xy + y = n(n 1)f
x2 x y y 2
(ax3 + by 3 )n
Let v = , then v is homogeneous of degree 3n, so
3n(3n 1)
2
2 v 2v 2
2 v (ax3 + by 3 )n
x 2
+ 2xy +y 2
= 3n(3n 1) = (ax3 + by 3 )n
x x y y 3n(3n 1)
Let w = xf ( xy ), then w is homogeneous of degree 1, so
2w 2w 2
2 w
x2 + 2xy + y = 1(1 1)w = 0
x2 x y y 2
Now u = v + w, so adding the above equations we have
2u 2u 2
2 u
x2 + 2xy + y = (ax3 + by 3 )n
x2 x y y 2

3
Question 2(a) 1. Without evaluating the involved integrals, show that
1
Z x Z
t dt x dt
+ =0
1 1 + t2 1 t(1 + t2 )
Z a+T
2. If f (x) is periodic of period T , show that f (t) dt is independent of a.
a

Solution.
du t 1 u2 u
1. Let t = u1 , so that dt = 2
and 2
= 2
= , implying that
u 1+t u 1+u 1 + u2
Z x Z 1   Z 1
t dt x u du x du
2
= 2
2
= 2
1 1+t 1 1+u u 1 u(1 + u )

Thus 1
Z x Z
t dt x dt
+ =0
1 1 + t2 1 t(1 + t2 )
Z a+T
2. Define F (a) = f (t) dt. We shall prove that F (a) = F (b) for any b. In particular
Ra T
F (a) = F (0) = 0
f (t) dt which is independent of a.

Z b+T Z a+T
F (b) F (a) = f (t) dt f (t) dt
b a
Z b+T Z a+T Z b
= f (t) dt f (t) dt f (t) dt
b b a
Z b+T Z b Z b
= f (t) dt + f (t) dt f (t) dt
b a+T a
Z b+T Z b
= f (t) dt f (t) dt
a+T a
Z b Z b
= f (u + T ) du f (t) dt u+T =t
a a
Z b Z b
= f (u) du f (t) dt = 0 f (u + T ) = f (u)
a a

Thus F (b) = F (a) for all b.

4
Question 2(b) Find the volume of the solid generated by revolving one arc of x = a(t
sin t), y = a(1 cos t) about its base.
Solution. One arc is given by 0 t 2 and the base is the x-axis. Thus
Z 2
dx
V = y 2 dt
dt
Z0 2 Z 2 
2 2 3 t 3
= a (1 cos t) a(1 cos t) dt = a 2 sin2 dt
0 0 2
Z
3 t
= 8a 2 sin6 d ( = )
0 2
Z
2
= 32a3 sin6 d (sin( ) = sin , so double the integral and half the limit)
0
531
= 32a3 = 5 2 a3
642 2

Z 2 Z x  21
Question 2(c) Evaluate (xy)2 +2(x+y)+1 dx dy by using the transformation
0 0
x = u(1 + v), y = v(1 + u). Assume u, v are positive in the region concerned.
Solution.
1. The Jacobian of the transformation is

(x, y) 1 + v u
= =1+u+v >0
(u, v) v 1 + u
because u 0, v 0.
2. The region of integration in the xy-plane is the triangle bounded by the lines y = 0, y =
x, x = 2.
3. The region of integration in the uv-plane lies in the first quadrant as u 0, v 0.
Clearly
x=y u=v
x = 2 u(1 + v) = 2
y = 0 v = 0 ( 1 + u > 0)

(1, 1)
v-axis
u(1 + v) = 2

(1, 0) (2, 0) u-axis

5
The curve u(1 + v) = 2 meets the u-axis at (2, 0) and the line u = v at (1, 1). The
region of integration is bounded by v = 0, v = u, u(1 + v) = 2 and therefor consists
of the triangle with vertices (0, 0), (1, 0), (1, 1) and the portion bounded by u = 1, v =
0, u(1 + v) = 2, in which u varies from 1 to 2, and v varies from 0 to 2u u
.

4.
  12   21
(x y)2 + 2(x + y) + 1 = (u v)2 + 2(2uv + u + v) + 1
1
= (u2 + v 2 + 2uv + 2u + 2v + 1) 2 = (1 + u + v)1

Thus the product of the Jacobian and the integrand is 1.

Thus
Z 2 Z x  21
2
I = (x y) + 2(x + y) + 1 dx dy
0 0
2
Z 1 Z u Z 2 Z
u
1
= dv du + dv du
0 0 1 0
Z 2
1 2
= + du 1
2 1 u
1
= 2 log 2
2

Paper II

Question 3(a) Find the maximum and minimum values of f (x, y) = 7x2 + 8xy + y 2 where
x, y are constrained by the relation x2 + y 2 = 1.

Solution. Let F (x, y) = 7x2 +8xy +y 2 +(x2 +y 2 1), where is Lagranges undetermined
multiplier. For extreme values
F F
= 14x + 8y + 2x = 0, = 8x + 2y + 2y = 0
x y

Thus (7 + )x + 4y = 0, 4x + (1 + )y = 0. Since (x, y) 6= (0, 0) because x2 + y 2 = 1, it


follows that

7 + 4
= (7 + )(1 + ) 16 = 2 + 8 9 = 0 = 9, 1
4 1 +

F F
x +y = 14x2 + 8xy + 2x2 + 8xy + 2y 2 + 2y 2 = 2f + 2(x2 + y 2 ) = 0
x y

6
Since x2 + y 2 = 1, we get f = at stationary points f = 9, 1. Thus the maximum
value of f is 9, minimum value is 1.
Check: We have found the maximum and minimum values without finding the stationary
points.
1 2
= 9 x = 2y, x2 + y 2 = 1 5y 2 = 1 y = , x =
5 5
1 2
= 1 y = 2x, x2 + y 2 = 1 5x2 = 1 x = , y =
5 5

In case (1), f (x, y) = 28


5
+ 165
+ 15 = 9. In case (2) f (x, y) = 57 16
5
+ 54 = 1, confirming the
above.
Note: The question could also be done by substituting x = cos t, y = sin t, and then
f (x, y) = 7 cos2 t+8 cos t sin t+sin2 t, which is now a function of one variable. Differentiating
and letting the derivative be 0, we get 14 cos t sin t + 8(cos2 t sin2 t) + 2 sin t cos t = 0.
Let z = tan t, then 2z 2 + 3z 2 = 0 z = 2, 21 (x, y) = ( 15 , 25 ), ( 25 , 15 )
f (x, y) = 9, 1.

You might also like